6
$\begingroup$

Let $a,q$ be co-prime integers and let $P(a,q)$ denote the set of primes congruent to $a$ modulo $q$. Is it known whether one can give an asymptotic formula for the expression

$$\displaystyle \sum_{\substack{n \leq x \\ p | n \Rightarrow p \in P(a,q)}} d(n),$$ where $d(n)$ is the number of divisors of $n$?

$\endgroup$

1 Answer 1

11
$\begingroup$

Sure. The generating function for the sum you want is the Dirichlet series $$ \sum_{\substack{ n=1\\p|n \implies p\equiv a\pmod q}}^{\infty} \frac{d(n)}{n^s} = \prod_{p\equiv a\pmod q} \Big(1- \frac{1}{p^s}\Big)^{-2}. $$ Using Dirichlet characters to isolate primes in progressions, you can express this as $$ \zeta(s)^{2/\phi(q)} \prod_{\chi \neq \chi_0 \pmod q} L(s,\chi)^{\overline{\chi(a)}/\phi(q)} G(s), $$ for a suitable Euler product $G$ converging absolutely to the right of $1/2$. Now use the Selberg-Delange method (see e.g. Tenenbaum's book). The asymptotic will be of the type $$ \sim C x(\log x)^{-1+2/\phi(q)} $$ for a suitable constant $C$.

$\endgroup$
4
  • $\begingroup$ Can the constant $ C $ be expressed in terms of $ a $ and $ q $? $\endgroup$ Sep 26, 2017 at 15:43
  • 1
    $\begingroup$ Yes, it will be $\prod_{\chi \neq \chi_0 \pmod q} L(1,\chi)^{\overline{\chi(a)}/\phi(q)} G(1)$ in the notation of the answer; $G$ can be explicitly written down easily. Tenenbaum's book has all the details. $\endgroup$ Sep 26, 2017 at 15:51
  • $\begingroup$ Greg forgot to divide by $\Gamma(2/\phi(q))$, but apart from that the constant is what he wrote. $\endgroup$
    – Lucia
    Sep 26, 2017 at 16:57
  • $\begingroup$ I think there is a $2$ missing in the exponent of the $L(s,\chi)$ term. $\endgroup$ Oct 20, 2018 at 11:23

Your Answer

By clicking “Post Your Answer”, you agree to our terms of service and acknowledge you have read our privacy policy.

Not the answer you're looking for? Browse other questions tagged or ask your own question.